Limits Or Not..

Calculus Level 5

Find the value of

β \beta = lim n r = 1 n ( 1 + r n ) 1 r \displaystyle\lim_{n\rightarrow \infty} \displaystyle \prod _{ r=1 }^{ n }{ { \left( 1+\dfrac { r }{ n } \right) }^{ \dfrac { 1 }{ r } } }

up to 3 decimal places.

Note: \prod _{}^{ } indicates the Product Series.


The answer is 2.276.

This section requires Javascript.
You are seeing this because something didn't load right. We suggest you, (a) try refreshing the page, (b) enabling javascript if it is disabled on your browser and, finally, (c) loading the non-javascript version of this page . We're sorry about the hassle.

3 solutions

Vraj Mehta
Jan 8, 2015

Solution \textbf{Solution}

β \beta = lim n r = 1 n ( 1 + r n ) 1 r \displaystyle\lim_{n\rightarrow \infty} \displaystyle \prod _{ r=1 }^{ n }{ { \left( 1+\dfrac { r }{ n } \right) }^{ \dfrac { 1 }{ r } } }

Taking Logarithm on Both Sides we get,

ln β \ln { \beta } = lim n r = 1 n 1 r ln ( 1 + r n ) \displaystyle\lim_{n\rightarrow \infty} \sum _{ r=1 }^{ n }{ \dfrac{1}{r}{ \ln\left( 1+\dfrac { r }{ n } \right) } }

Multiplying And Dividing By n \textbf{n} ,We get

ln β \ln { \beta } = lim n r = 1 n 1 n n r ln ( 1 + r n ) \displaystyle\lim_{n\rightarrow \infty} \sum _{ r=1 }^{ n }{ \dfrac{1}{n} \dfrac{n}{r}{ \ln \left( 1+\dfrac { r }{ n } \right) } }

Which Can be Rewritten as,

ln β \ln { \beta } = lim n r = 1 n 1 n 1 ( r n ) ln ( 1 + r n ) \displaystyle\lim_{n\rightarrow \infty} \sum _{ r=1 }^{ n }{ \dfrac{1}{n} \dfrac{1}{(\dfrac{r}{n})}{ \ln \left( 1+\dfrac { r }{ n } \right) } }

Now It can Be Written in Form of Integration As Using Riemann sums \textbf{Riemann sums} ,

ln β \ln { \beta } = 0 1 ln ( 1 + x ) x d x \displaystyle\int_{0 }^{1}{\dfrac{\ln(1+x)}{x}} \mathrm{d}x

Using Series Expansion Since x 1 |x| \leq 1 , We Get,

ln β \ln { \beta } = 0 1 n = 1 ( 1 ) n + 1 n x n x d x \displaystyle\int_{0 }^{1}{\dfrac{\displaystyle \sum _{ n=1 }^{ \infty } \dfrac { (-1)^{ n+1 } }{ n } x^{ n }}{x}} \mathrm{d}x

Since,

ln ( 1 + x ) = n = 1 ( 1 ) n + 1 n x n = x x 2 2 + x 3 3 f o r x 1 , u n l e s s x = 1. \ln { ( } 1+x)=\displaystyle \sum _{ n=1 }^{ \infty } \dfrac { (-1)^{ n+1 } }{ n } x^{ n }=x-\dfrac { x^{ 2 } }{ 2 } +\dfrac { x^{ 3 } }{ 3 } -\cdots \\ \quad { for }\quad \left| x \right| \leq 1,\quad { unless }\quad x=-1.

Which Can Be Simplified to,

ln β \ln { \beta } = 0 1 x x 2 2 + x 3 3 x d x \displaystyle\int_{0 }^{1}{ \dfrac{x-\dfrac { x^{ 2 } }{ 2 } +\dfrac { x^{ 3 } }{ 3 } -\cdots}{x}} \mathrm{d}x

Which Again Simplifies to,

ln β \ln { \beta } = 0 1 1 x 2 + x 2 3 d x \displaystyle \int_{0 }^{1}1-\dfrac { x }{ 2 } +\dfrac { x^{2} }{ 3 } -\cdots\mathrm{d}x

Integrating We Get,

ln β \ln { \beta } = x x 2 2 2 + x 3 3 2 x 4 4 2 + 0 1 \displaystyle \left. x - \dfrac { x^{ 2 } }{ 2^{2} } +\dfrac { x^{ 3 } }{ 3^{2} } - \dfrac { x^{ 4} }{ 4^{2}} +\cdots \right|_0^1

Putting The Limits We Get,

ln β \ln { \beta } = 1 1 2 2 + 1 3 2 1 4 2 + \displaystyle 1 - \dfrac { 1 }{ 2^{2} } +\dfrac {1 }{ 3^{2} } - \dfrac {1}{ 4^{2}} +\cdots

And It is Known That

n = 1 ( 1 ) n + 1 n 2 \displaystyle \sum _{ n=1 }^{ \infty } \dfrac { (-1)^{ n+1 } }{ n^{2} } = π 2 12 \dfrac{\pi^{2}}{12}

So

ln β \ln { \beta } = π 2 12 \dfrac{\pi^{2}}{12}

And

β \beta = e π 2 12 { \mathrm{e} }^{ \dfrac{\pi ^{ 2}}{12} }

Which Is Approximately

β \beta = 2.27610.... \textbf{2.27610....}

Another way of evaluating the integral - 0 1 log ( 1 + x ) x d x = 0 1 0 1 1 1 + x y d x d y = 0 1 0 1 n = 0 ( 1 ) n x n y n d x d y = n = 0 0 1 0 1 ( 1 ) n x n y n d x d y = n = 0 ( 1 ) n ( n + 1 ) 2 \begin{aligned} \int_{0}^{1} \dfrac{\log(1+x)}{x} \mathrm{d}x&=\int_{0}^{1} \int_{0}^{1}\dfrac{1}{1+xy} \mathrm{d}x \mathrm{d}y\\ &=\int_{0}^{1} \int_{0}^{1} \sum_{n=0}^{\infty}(-1)^nx^ny^n \mathrm{d}x \mathrm{d}y\\ &= \sum_{n=0}^{\infty} \int_{0}^{1} \int_{0}^{1} (-1)^nx^ny^n \mathrm{d}x \mathrm{d}y\\ &=\sum_{n=0}^{\infty} \dfrac{(-1)^n}{(n+1)^2} \end{aligned}

Pratik Shastri - 6 years, 5 months ago

Log in to reply

Nice Way Of Integration.

Vraj Mehta - 6 years, 5 months ago

I Am Hoping For Some one to give me the proof that

n = 1 ( 1 ) n + 1 n 2 \displaystyle \sum _{ n=1 }^{ \infty } \dfrac { (-1)^{ n+1 } }{ n^{2} } = π 2 12 \dfrac{\pi^{2}}{12}

Vraj Mehta - 6 years, 5 months ago

Log in to reply

n = 1 ( 1 ) n + 1 n 2 = n > 0 , odd 1 n 2 n > 0 , even 1 n 2 = ( ζ ( 2 ) ζ ( 2 ) 4 ) ζ ( 2 ) 4 = ζ ( 2 ) 2 \begin{aligned}\sum_{n=1}^{\infty} \dfrac{(-1)^{n+1}}{n^2}&=\sum_{n>0, \ \text{odd}} \dfrac{1}{n^2}-\sum_{n>0, \ \text{even}} \dfrac{1}{n^2}\\ &= \left(\zeta(2)-\dfrac{\zeta(2)}{4}\right)-\dfrac{\zeta(2)}{4}\\ &=\boxed{\dfrac{\zeta(2)}{2}} \end{aligned}

Facts used

1 ) n > 0 , even 1 n 2 = ζ ( 2 ) 4 1) \ \ \displaystyle\sum_{n>0, \ \text{even}} \dfrac{1}{n^2}=\dfrac{\zeta(2)}{4}

2 ) n > 0 , odd 1 n 2 = ζ ( 2 ) n > 0 , even 1 n 2 2) \ \ \displaystyle\sum_{n>0, \ \text{odd}} \dfrac{1}{n^2}=\zeta(2)-\displaystyle\sum_{n>0, \ \text{even}} \dfrac{1}{n^2}

You may include this in your solution if you want.

Pratik Shastri - 6 years, 5 months ago

Log in to reply

Thanks For The Proof.

Vraj Mehta - 6 years, 5 months ago

Log in to reply

@Vraj Mehta Are both of you in same class?

Krishna Sharma - 6 years, 5 months ago

Log in to reply

@Krishna Sharma Yes . Same class.

Pratik Shastri - 6 years, 5 months ago

Log in to reply

@Pratik Shastri Do you know Hitarth Shah - he also studies in DPS and goes to fiitjee

U Z - 6 years, 5 months ago

Log in to reply

@U Z No, not in our school.

Pratik Shastri - 6 years, 5 months ago

nice ques had fun while solving :P

A Former Brilliant Member - 4 years, 5 months ago

did the same way!!!!

A Former Brilliant Member - 3 years, 7 months ago
Rohit Shah
Jan 7, 2015

Take logarithm on both sides. Sum the series using integration and evaluate the integral by using zeta function. Take antilog and that's the answer

I did it that way , but I am getting 4 e \dfrac{4}{e}

β = b \beta = b

l n b = 1 n l i m n l o g ( 1 + r n ) lnb = \dfrac{1}{n}lim_{n \to \infty} \sum log(1 + \dfrac{r}{ n})

l n b = 0 1 l o g ( 1 + x ) d x \displaystyle lnb = \int_{0}^{1} log(1 + x) dx

l n b = l n 4 l n e = l n 4 e lnb = ln4 - lne = ln\dfrac{4}{e}

β = 4 e = 1.471 \beta = \dfrac{4}{e} = 1.471

U Z - 6 years, 5 months ago

Log in to reply

Its

lim n r = 1 n ( 1 + r n ) 1 r \displaystyle\lim_{n\rightarrow \infty} \displaystyle \prod _{ r=1 }^{ n }{ { \left( 1+\dfrac { r }{ n } \right) }^{ \dfrac { 1 }{ r } } }

And

Not

lim n r = 1 n ( 1 + r n ) 1 n \displaystyle\lim_{n\rightarrow \infty} \displaystyle \prod _{ r=1 }^{ n }{ { \left( 1+\dfrac { r }{ n } \right) }^{ \dfrac { 1 }{ n } } }

So The problem is Right.

Vraj Mehta - 6 years, 5 months ago

Log in to reply

Sorry , I reported it , thanks a nice problem

U Z - 6 years, 5 months ago

Log in to reply

@U Z Its Okay,I Posted the Solution Now..

Vraj Mehta - 6 years, 5 months ago

it was good that i remembered some expansions and special series and suggest you remember them too ! :) taylor and basel problem types

0 pending reports

×

Problem Loading...

Note Loading...

Set Loading...